If y + 4 = (x - 2)², x + 4 = (y - 2)², and x ≠ y, what is the value of x² + y²? (2015 AMC 10A #16)
(A) 10 (B) 15 (C) 20 (D) 25 (E) 30
The sum of an infinite geometric sequence is S, and the second term in the series is 1. What is the smallest possible value of S? (2016 AMC 10B #16)
(A) (1+√5)/2 (B) 2 (C) √5 (D) 3 (E) 4
Four cubes with edge lengths 1, 2, 3, and 4 are stacked as shown. What portion of XY is contained in the cube with edge length 3? (2014 AMC 10A #19)
(A) (3√33)/5 (B) 2√3 (C) (2√33)/5 (D) 4 (E) 3√2
For how many values of k do the graphs of x² + y² = k² and k = xy never intersect? (2010 AMC 12A #13)
(A) 0 (B) 1 (C) 2 (D) 4 (E) 8
Let N be a positive multiple of 5. One red ball and N green balls are arranged in a line in random order. Let P(N) be the probability that at least 3/5 of the green balls are on the same side of the red ball. Observe that P(5) = 1 and that P(N) approaches 4/5 as N grows large. What is the sum of the digits of the least value of N such that P(N) < 321/400? (2016 AMC 10A #17)
(A) 12 (B) 14 (C) 16 (D) 18 (E) 20
For some particular value of N, when (a+b+c+d+1)^N is expanded and like terms are combined, the resulting expression contains exactly 1001 terms that include all four variables a, b, c, d, each to some positive power. What is N ? (2016 AMC 10A #20)
(A) 9 (B) 14 (C) 16 (D) 17 (E) 29
A quadrilateral is inscribed in a circle of radius 200√2. Three of the sides of this quadrilateral have length 200. What is the length of the fourth side? (2016 AMC 10A #24)
(A) 200 (B) 200√2 (C) 200√3 (D) 300√2 (E) 500
How many ordered triples (x, y, z) of positive integers satisfy lcm(x,y) = 72, lcm(x,z) = 600, and lcm(y, z) = 900? (2016 AMC 10A #25)
(A) 15 (B) 16 (C) 24 (D) 17 (E) 64
For how many integers n between 1 and 50, inclusive, is (n² - 1)!/(n!)ⁿ an integer? (Recall that 0! = 1.) (2019 AMC 10A #25)
(A) 31 (B) 32 (C) 33 (D) 34 (E) 35
_ _ _ _ _ _ _ _ _ _ _ _ _ _ _ _ _ _ _ _ _ _ _ _ _ _ _ _ _ _ _ _ _ _ _ _ _ _ _ _ _ _ _ _ _ _ _ _ _ _ _ _ _ _ _ _ _ _ _ _ _ _ _ _ _ _ _ _ _ _ _ _ _ _ _ _ _ _ _ _ _ _ _ _ _ _ _ _ _ _ _ _ _ _ _ _ _ _ _ _ _ _ _ _ _ _ _ _ _ _ _ _ _ _ _ _ _ _ _ _ _ _ _ _ _ _ _ _ _ _ _ _ _ _ _ _ _ _ _ _ _ _ _ _
Solution 1 (Fast): Expanding the equations and simplifying, we get y + 4x = x², and x + 4y = y². We can add the two equations and subtract the first from the second, and when we simplify, we get 5(x + y) = x² + y², and 3(y - x) = y² - x². Factoring y² - x² as a difference of squares and simplifying, we get y + x = 3. Plugging this into the first equation (5(x + y) = x² + y²), we get x² + y² = 5(3) = 15. Thus, our desired answer is (B).
~ Muhammad Sheriff
Solution 1 (Quadratics): Since the geometric sequence needs to have a constant ratio of less than 1 (or else the sum would be infinite), we can use the sum of an infinite geometric series formula which is a / (1 - r) where a is the first term and r is the constant ratio. We can set up an equation to find S which is (1 / r) / (1-r) because 1/r is the first term. We can simplify the equation into 1 / (r - r²). Now, we can set this equation to be equal to S. This means r - r² is equal to 1/S. Next, we can use the quadratic formula to find the constant ratio of r based on S. R is equal to (1 ∓ √(1 - 4(1 / S)))/2. Since the constant ratio has to be a real number, the least possible value of S must be 4 because (1 - 4( 1 / S)) must not be below zero or else we would get a non-real number. Thus, the answer is (E).
~ Roger Huynh
Solution 2 (Easy): We can use algebra to label the first term of the geometric series as x. We can then see that our common ratio is 1/x. Using the infinite geometric series summation formula, the sum of the series is x / ((x - 1)/ x). This simplifies to x² / (x - 1). We can see that the smallest possible value of this expression is when x - 1 = 1 (dividing by something less than one is multiplying by something greater than one). Thus, the smallest possible value of S is 4, or (E).
~Muhammad Sheriff
Solution 1 (Vectors): We can use a "vector" to model XY . We first need to create our coordinate system. Let the bottom right corner of the cube of edge length 4 be the origin. Thus, XY has endpoints at (4,0,0) and (0,4,10). We can now see that if x changes by -4, y changes by 4 and z changes by 10. We can model this as such: x changes by -1 ➨ y changes by 1 & z changes by 5/2 (this also works vice versa and etc). We need to find the coordinates of segment XY when z is 4 and 7. To make z = 4, z needs to change by 4, which means x changes by -8/5 and y changes by 8/5. Thus, the coordinates of the point contained in the bottom layer of the cube with side length 3 is (4 - 8/5, 0 + 8/5, 0 + 4) = (-12/5, 8/5, 4). Using this same strategy, we find the coordinates of the point in which line segment XY intersects is (-6/5, 14/5, 7). Using the 3D Pythagorean Theorem to find the distance between the two points, we get our desired answer, which is (A).
~Muhammad Sheriff
Solution 2 (Quick): We can calculate the distance from X to Y using 3 dimensional Pythagorean Theorem. The distance between X and Y is √(4² + 4² + 10²) which equals to 2√33. The square with a side length of three contains 3/10 of the distance. So, the distance contained by the side length 3 cube is (3√33)/5. The answer is (A).
~Roger Huynh
Solution 1 (Factoring): We can see that we have to find the number of unique values of xy such that x²+y² = (xy)² has no solutions. We can move all terms to one side and factor the expression as (x² - 1)(y² - 1) = 1. We can see that as long as we set a value of x or y that is not 1 or -1, there is always a value of the other variable such that the expression is 1. Thus, the only ordered pairs that work are (x,y) = (1,1), (1,-1), (-1,1), and (-1,-1). The only unique values for the product of xy is 1 and -1. Thus, our desired answer is 2, or (C).
~Muhammad Sheriff
Solution 2 (Quick): We can test out a few values to figure out that |k| has to be smaller than 2 (If k = 2 then the circle and k = xy graph will intersect at (√2, √2). Thus, the only values of k that fits the condition are 1 and -1. (k can't be zero or else both graphs will intersect at (0,0)). This makes our desired answer is 2, or (C).
~Roger Huynh
Solution 1 (Algebra): Let N = 5x for some integer x > 1. We can observe that the only values that work are "3x Green Balls, 1 Red Ball, 2x Green Balls", "3x + 1 Green Balls, 1 Red Ball, 2x - 1 Green Balls", "3x + 2 Green Balls, 1 Red Ball, 2x - 2 Green Balls"..., "5x Green Balls, 1 Red Ball, 0 Green Balls" and "2x Green Balls, 1 Red Ball, 2x Green Balls", "2x - 1 Green Balls, 1 Red Ball, 3x + 1 Green Balls", "2x - 2 Green Balls, 1 Red Ball, 3x + 2 Green Balls"..., "0 Green Balls, 1 Red Ball, 5x Green Balls". The first & second "case" has 2x + 1 permutations that work, and there are a total of 5x + 1 permutations. This makes the probabilty (4x + 2) / (5x + 1). We want to find the least possible value of x that satisfies (4x + 2) / (5x + 1) < 321/400. Multiplying both sides by (5x + 1)(400) and simplifying, we get 1600x + 800 < 1605x + 321, so 5x > 479. The least possible value of 5x that satisfies this is 480 since x has to be an integer, so x is 96. Since N = 5x, the least possible value of N that satisfies the inequality is 480. Thus, our desired answer is (A).
~Muhammad Sheriff
Solution 2 (Pattern recognition and Equations): We know that there are N+1 ways to place the red ball. The question asks for the least value of N such that P(N) is less than 321/400. If N = 5, P(N) = 1. If N = 10, P(N) = 10/11. If N=15, P(N) = 14/16. And if N = 20, P(N) = 18/21. If we continue this pattern, we can find out that P(N) = 1 - (N/5)-1)/(N+1) which is equal to (N(4/5)+2)/(N+1). Let's set this value to 321/400. Now, we have an equation of (N(4/5)+2)/(N+1) = 321/400. Multiply both sides by N+1 and you get N(4/5) + 2 = (321N + 321)/400. Now we multiply both sides by 400 and we get an equation of 320N + 800 = 321N + 321. Subtract both sides by 320N + 321 and we know that N = 479 if P(N) is equal to 321/400. For P(N) to be smaller, N must be larger so N must be equal to 480. Thus, the answer is (A).
~Roger Huynh
Solution 1 (Stars & Bars): Let us first derive the formula for the amount of terms that don't have to include all four variables, then we can update it to include these constraints. We can see that the maximum sum of the powers of a given term can be N, and the minimum is 0 (if the term is 1), with all values in between being possible. Thus, we can see that the number of terms is the amount of ordered pairs (a, b, c, d) that satisfy a +b +c +d <= N. Since the term has to have abcd in it, we can just modify the inequality to be a +b +c +d <= N - 4 (Since there are N - 4 powers to "choose" from). To solve this, we can use stars and bars with an additional variable x that will be the "distance" required to get to 5. To find the number of quintuples, we can just use the non-negative stars and bars formula, and this is equal to nCr(4 + N - 4,4) = nCr(N, 4). This value is equal to 1001, and we can just test out values to get N as 14, so our final answer is (B).
~Muhammad Sheriff
Solutions 2 (Multinomial Expansion): Using the Multinomial Expansion, in the expansion of (x1 + x2 + x3 + ... + xm)^n, each term corresponds to an exponent combination (e1, e2, e3, ... em) = n and the number of such terms is n + m - 1 choose m - 1. Now, we need to use change of variables because the multinomial expansion only works for variables larger or equal to 0, not 1. We can define a' = a - 1, b' = b - 1, c' = c -1, d' = d - 1. So now, a', b', c', d' ≧ 0. Since a + b + c + d + e = N, we get (a' + 1) + (b' + 1) + (c' + 1) + (d'+1) + e = N ⇒ a' + b' + c' + d' + e = N - 4. Now that all variables are ≧ 0, we can apply the formula. Number of solutions = ((N-4) + 5 - 1 choose 5 - 1) = (N choose 4). We can now make an equation which is (N * (N-1) * (N-2) * (N-3))/24 = 1001. This means that (N * (N-1) * (N-2) * (N-3)) = 24024. We can factorize 24024 into 2^3 * 3 * 7 * 11 * 13. We can then figure out that the four integers are 11, 12, 13, and 14. Thus, N = 14. (A) is the answer.
~Roger Huynh
Solution 1 (Trigonometry): In the diagram above, let O be the center of the circle and ABCD be the quadrilateral. Drawing OA, OB, OC, and OD, we create three congruent isosceles triangles. Let θ be the center of the circle. Using the Law of Cosines, √(2 ⋅ (200√2)²-2 ⋅ 200√2 ⋅ 200√2 cos(θ)) = 200. Simplifying, we get √(160000 - 160000 ⋅ cos(θ)) = 200, or 1/2 = √(1 - cos(θ)), which leads to θ = arccos(3/4). To find AD, we can use the Law of Cosines once more, yielding √(160000 - 160000 ⋅ cos(3 ⋅ arccos(3/4)) = AD, which leads to AD/400 = √(1 - cos(3 ⋅ arccos(3/4)). To simplify this, we can use the triple cosine formula, which states that cos(3x) = 4 ⋅ cos³(x) - 3 ⋅ cos(x). Substituting this in, we get cos(3 ⋅ arccos(3/4) = 4 ⋅ (cos³(arccos(3/4)) - cos(arccos(3/4)) which when we simpify, we get 27/16 - 9/4 = -9/16. When we substitute this back it, we get √(1 - (- 9/16)) = 5/4 = AD/400. Multiplying both sides by 400, we get AD = 500, so our desired answer is (E).
~Muhammad Sheriff
Solution 1 (Casework): We can observe that x has to be 2^a ⋅ 3^b, y has to be 2^c ⋅ 3^d, and z has to be 2^e ⋅ 3^f ⋅ 5^2 such that (a,b,c,d,e,f) satisfy:
max(e,c) = 2
max(f,d) = 2
max(a,c) = 3
max(b,d) = 2
max(a,e) = 3
max(b,f) = 1
From equation 4 & 6, we can see that d = 2 (otherwise, b would have to equal 2 which is a contradiction to equation 6). Thus, equation 2 does not matter anymore (since f is anyway less than or equal to 1). Now, we can do casework on e.
CASE 1: e = 2: If e is 2, then there are 3 possibilities for the ordered triple (e,a,c) which are (3,2,0), (3,2,1), and (3,2,2).
CASE 2: e != 2: If e is not 2, then c has to be 2, and there are 3 possibilities for the ordered triple (e,a,c) which are (3,0,2), (3,1,2), and (3,2,2).
By the principle of inclusion-exclusion, to find the number of unique possibilities for the ordered triple (e,a,c), we have to the number of cases for each and subtract the intersection, and since the only possibility for their intersection is the ordered triple (3,2,2), the number of unique possibilities is 3 + 3 - 1 = 5. Since for each of these cases, there is 3 ordered pairs for b & f which are (0,1), (1,0), and (1,1), our desired answer is 3 ⋅ 5 = 15, or (A).
~Muhammad Sheriff
Solution 1 (Fast): We can show using Legendre's Formula that if (n² - 1)! is divisible by nⁿ, then it is also divisible by (n!)ⁿ. We can see that if n is prime, then the statement holds false (since n² - 1 < n ⋅ n). Since there are 15 primes from 1 to 50, there are at least 15 numbers that do not hold. We can see that we only now need to check the non-primes from 1 to 10 (as n gets larger, it will always hold for non-primes). We can see that 4 is the only non-prime that does not hold. Thus, there are 16 numbers from 1 to 50 that do not hold to the statement, and our desired answer is 50 - 16, or (D).
~Muhammad Sheriff